Загрузка страницы

Variational Principle in QM Example 1: Quantum Harmonic Oscillator

In this video I will do an example for the variational principle in QM. I will estimate the ground state for the quantum harmonic oscillator.

My name is Nick Heumann, I am a recently graduated physicist. I love to teach physics, so I decided to give YouTube a try. English is not my first language, but I hope that you can understand me well enough regardless.
▬ Contents of this video ▬▬▬▬▬▬▬▬▬▬
00:00 Explaining the problem
01:28 Normalizing the wavefunction
01:50 How to solve a gaussian integral (EXTREMELY IMPORTANT)
03:55 Calculating expectation value of kinetic part
07:10 Another EXTREMELY important integral
12:50 Expectation value of the potential
17:30 Minimizing the parameter

Видео Variational Principle in QM Example 1: Quantum Harmonic Oscillator канала Nick Heumann
Показать
Комментарии отсутствуют
Введите заголовок:

Введите адрес ссылки:

Введите адрес видео с YouTube:

Зарегистрируйтесь или войдите с
Информация о видео
24 февраля 2022 г. 5:47:57
00:21:46
Другие видео канала
Griffiths Problem 4.4 (3rd ed) Solution: The OTHER Solution to the Angular EquationGriffiths Problem 4.4 (3rd ed) Solution: The OTHER Solution to the Angular EquationGriffiths QM Problem 4.8: Infinite Spherical Well for l=1Griffiths QM Problem 4.8: Infinite Spherical Well for l=1Proving that the solutions to the Schrödinger Equation can be separated into even and odd solutionsProving that the solutions to the Schrödinger Equation can be separated into even and odd solutionsThe EASIEST Explanation of the Gram-Schmidt Procedure (Griffiths QM 3rd ed. Problem 3.4)The EASIEST Explanation of the Gram-Schmidt Procedure (Griffiths QM 3rd ed. Problem 3.4)Griffiths Intro to QM - Problem 8.15: The ULTIMATE WKB Problem -HARDEST ONE YETGriffiths Intro to QM - Problem 8.15: The ULTIMATE WKB Problem -HARDEST ONE YETThe Integral You HAVE TO KNOW to MASTER Quantum Mechanics!The Integral You HAVE TO KNOW to MASTER Quantum Mechanics!How do LASERs work? A Quantum Mechanical Explanation Using Perturbation TheoryHow do LASERs work? A Quantum Mechanical Explanation Using Perturbation TheoryHow to determine the 4 Wavefunctions of the Hydrogen Atom for n=2How to determine the 4 Wavefunctions of the Hydrogen Atom for n=2How To Go From Arcsin(x) to Ln (natural logarithm) (Arcsin - Log Formulation)How To Go From Arcsin(x) to Ln (natural logarithm) (Arcsin - Log Formulation)Griffiths QM 6.26 (3rd ed) 6.28 (2nd ed): Hyperfine Splitting for Muonic H, Positronium, MuoniumGriffiths QM 6.26 (3rd ed) 6.28 (2nd ed): Hyperfine Splitting for Muonic H, Positronium, MuoniumGriffiths QM 2.27 Solution: Finding Transmission coefficient for double delta potentialGriffiths QM 2.27 Solution: Finding Transmission coefficient for double delta potentialThe WKB Approximation for Energy less than V - Finding the Transmission coefficientThe WKB Approximation for Energy less than V - Finding the Transmission coefficientWhy we need QFT & Derivation of Klein-Gordon Langriangian DensityWhy we need QFT & Derivation of Klein-Gordon Langriangian DensityHow to Solve The Infinite Spherical Well (FULLY EXPLAINED)How to Solve The Infinite Spherical Well (FULLY EXPLAINED)Griffiths QM 6.13 (3d edition) Solution: Relativistic Correction for Quantum Harmonic OscillatorGriffiths QM 6.13 (3d edition) Solution: Relativistic Correction for Quantum Harmonic OscillatorWKB Approximation Connection Formulas Example: Potential Well with no Vertical WallsWKB Approximation Connection Formulas Example: Potential Well with no Vertical WallsGriffiths QM 4.15 Solution: Proving Radial Wavefunction and finding expectation values for l=n-1Griffiths QM 4.15 Solution: Proving Radial Wavefunction and finding expectation values for l=n-1Solving the Schrödinger Equation for the Infinite Potential Well (Particle in a square box)Solving the Schrödinger Equation for the Infinite Potential Well (Particle in a square box)Griffiths QM Problem 2.37 (3rd edition): Find expectation values for nth stationary state of QHOGriffiths QM Problem 2.37 (3rd edition): Find expectation values for nth stationary state of QHOGriffiths Intro to QM 7.11 (3rd edition) solution: ground state energy of H using Ae^(-br^2)Griffiths Intro to QM 7.11 (3rd edition) solution: ground state energy of H using Ae^(-br^2)
Яндекс.Метрика